LSAT and Law School Admissions Forum

Get expert LSAT preparation and law school admissions advice from PowerScore Test Preparation.

 Administrator
PowerScore Staff
  • PowerScore Staff
  • Posts: 8917
  • Joined: Feb 02, 2011
|
#40386
Complete Question Explanation
(The complete setup for this game can be found here: lsat/viewtopic.php?t=13080)

The correct answer choice is (C)

Justify questions, which appear infrequently in the Logic Games section, require you to select an answer choice that, when added to the rest of the rules, forces the game to have only one possible solution. Our templates would be tremendously helpful in this regard. Here’s why:

Answer choice (A) is incorrect, because if Q is shown third, then S and T can alternate between positions 1 and 5 (Template 1).

Answer choice (B) is incorrect, because if R is first, then S and Q can alternate between positions 2 and 4 (Template 2.1).

Answer choice (C) is the correct answer choice. If S is shown second, then only Template 2.1. would provide an appropriate solution. Q would be forced into position 4, ensuring that the order in which the houses are shown is completely determined:
PT72_Game_#2_#8_diagram 1.png
Answer choice (D) is incorrect, because T is fifth in Templates 2.1 as well as 2.2.

Answer choice (E) is incorrect. Although only Template 1 allows for V to be shown fourth, note that the exact positions of S and T are not determined in that template.
You do not have the required permissions to view the files attached to this post.
 angelsfan0055
  • Posts: 39
  • Joined: Feb 26, 2021
|
#87518
What is generally the best approach to justify questions? I've found I am consistently getting these wrong and in a section like this one with a pattern game, would like to have all the points I can get.
 Robert Carroll
PowerScore Staff
  • PowerScore Staff
  • Posts: 1787
  • Joined: Dec 06, 2013
|
#87566
angels,

Information on Justify questions in general can be found here:

viewtopic.php?p=86790#p86790

viewtopic.php?p=87420#p87420

viewtopic.php?p=87417#p87417

In this particular case, because the best approach to the initial diagram uses templates, we should exploit the templates to answer this question, as given in the first post.

Robert Carroll

Get the most out of your LSAT Prep Plus subscription.

Analyze and track your performance with our Testing and Analytics Package.